• Shuffle
    Toggle On
    Toggle Off
  • Alphabetize
    Toggle On
    Toggle Off
  • Front First
    Toggle On
    Toggle Off
  • Both Sides
    Toggle On
    Toggle Off
  • Read
    Toggle On
    Toggle Off
Reading...
Front

Card Range To Study

through

image

Play button

image

Play button

image

Progress

1/10

Click to flip

Use LEFT and RIGHT arrow keys to navigate between flashcards;

Use UP and DOWN arrow keys to flip the card;

H to show hint;

A reads text to speech;

10 Cards in this Set

  • Front
  • Back
A 70yo M with h/o HTN and DM presents with a 2-mo h/o increasing paroxysmal nocturnal dyspnea and SOB with minimal exertion. An echocardiogram shows an ejection fraction of 25%. Which one of the patients current medications should be discontinued?
A. Lisinopril (Zestril)
B. Pioglitazone (Actos)
C. Glipizide (Glucotrol)
D. Metoprolol (Toprol-XL)
Pioglitzaone

thiazolidinediones (TZDs) are associated with fluid retention, and their use can be complicated by the development of heart failure. Caution is necessary when prescribing TZDs in patients with known heart failure or other heart diseases, those with preexisting edema, and those on concurrent insulin therapy
What is the most common cause of hypertension in children under 6 years of age?
Renal Parenchymal Disease

The most common cause of hypertension is renal parenchymal disease, and a urinalysis, urine culture, and renal ultrasonography should be ordered for all children presenting with hypertension.
A 72-year-old African-American male with New York Heart Association Class III heart failure sees you for follow-up. He has shortness of breath with minimal exertion. The patient is adherent to his medication regimen. His current medications include lisinopril (Prinivil, Zestril), 40 mg twice daily; carvedilol (Coreg), 25 mg twice daily; and furosemide (Lasix), 80 mg daily. His blood pressure is 100/60 mm Hg, and his pulse rate is 68 beats/min and regular. Findings include a few scattered bibasilar rales on examination of the lungs, an S3 gallop on examination of the heart, and no edema on examination of the legs. An EKG reveals a left bundle branch block, and echocardiography reveals an ejection fraction of 25%, but no other abnormalities. What's the appropriate next step?
Refer for cardiac resynchronization therapy (CRT)

Note: he's already on maximum doses of ACEI, loop diuretic, beta-blocker

Using a pacemaker-like device, CRT aims to get both ventricles contracting simultaneously, overcoming the delayed contraction of the left ventricle caused by the left bundle-branch block. These guidelines were refined by an April 2005 AHA Science Advisory, which stated that optimal candidates for CRT have a dilated cardiomyopathy on an ischemic or nonischemic basis, an LVEF ≤0.35, a QRS complex ≥120 msec, and sinus rhythm, and are NYHA functional class III or IV despite maximal medical therapy for heart failure.
What dietary change recommended for the prevention and treatment of cardiovascular disease has been shown to decrease the rate of sudden death?
increase intake of omega 3 fatty acids

Omega-3 fats contribute to the production of eicosapentaenoic acid (EPA) and docosahexaenoic acid (DHA), which inhibit the inflammatory immune response and platelet aggregation, are mild vasodilators, and may have antiarrhythmic properties. The American Heart Association guidelines state that omega-3 supplements may be recommended to patients with preexisting disease, a high risk of disease, or high triglyceride levels, as well as to patients who do not like or are allergic to fish. The Italian GISSI study found that the use of 850 mg of EPA and DHA daily resulted in decreased rates of mortality, nonfatal myocardial infarction, and stroke, with particular decreases in the rate of sudden death.
A 75-year-old male presents to the emergency department with a several-hour history of back pain in the interscapular region. His medical history includes a previous myocardial infarction (MI) several years ago, a history of cigarette smoking until the time of the MI, and hypertension that is well controlled with hydrochlorothiazide and lisinopril (Prinivil, Zestril). The patient appears anxious, but all pulses are intact. His blood pressure is 170/110 mm Hg and his pulse rate is 110 beats/min. An EKG shows evidence of an old inferior wall MI but no acute changes. A chest radiograph shows a widened mediastinum and a normal aortic arch, and CT of the chest shows a dissecting aneurysm of the descending aorta that is distal to the proximal abdominal aorta but does not involve the renal arteries. Which one of the following would be the most appropriate next step in the management of this patient?
Dx: aortic dissection
next step: Intravenous labetalol (Normodyne, Trandate)

Initial management should reduce the systolic blood pressure to 100-120 mm Hg or to the lowest level tolerated. The use of a β-blocker such as propranolol or labetalol to get the heart rate below 60 beats/min should be first-line therapy. If the systolic blood pressure remains over 100 mm Hg, intravenous nitroprusside should be added. Without prior beta-blocade, vasodilation from the nitroprusside will induce reflex activation of the sympathetic nervous system, causing increased ventricular contraction and increased shear stress on the aorta.
According to the U.S. Preventive Services Task Force, what are the screening recommendations for an abdominal aortic aneurysm?
The guideline recommends one-time screening with ultrasonography for AAA in men 65-75 years of age who have ever smoked. No recommendation was made for or against screening women. Men with a strong family history of AAA should be counseled about the risks and benefits of screening as they approach 65 years of age.
A 36-year-old white female presents to the emergency department with palpitations. Her pulse rate is 180 beats/min. An EKG reveals a regular tachycardia with a narrow complex QRS and no apparent P waves. The patient fails to respond to carotid massage or to two doses of intravenous adenosine (Adenocard), 6 mg and 12 mg. The most appropriate next step would be to administer intravenous
verapamil (Calan)

If supraventricular tachycardia is refractory to adenosine or rapidly recurs, the tachycardia can usually be terminated by the administration of intravenous verapamil or a β-blocker. If that fails, intravenous propafenone or flecainide may be necessary. It is also important to look for and treat possible contributing causes such as hypovolemia, hypoxia, or electrolyte disturbances. Electrical cardioversion may be necessary if these measures fail to terminate the tachyarrhythmia.
The blood pressure goal for a patient who has uncomplicated diabetes mellitus is
BP goal: 130/80mmHg

Aggressive control of blood pressure to <135/85 mm Hg in hypertensive patients and to <130/80 mm Hg in diabetic patients is recommended. Lowering blood pressure may reduce stroke rates by 40%-52% and cardiovascular morbidity by 18%-20%
A 60-year-old African-American female has a history of hypertension that has been well controlled with hydrochlorothiazide. However, she has developed an allergy to the medication. Successful monotherapy for her hypertension would be most likely with which one of the following?
A. Lisinopril (Prinivil, Zestril)
B. Hydralazine (Apresoline)
C. Clonidine (Catapres)
D. Atenolol (Tenormin)
E. Diltiazem (Cardizem)
Diltiazem (Cardizem)

Monotherapy for hypertension in African-American patients is more likely to consist of diuretics or calcium channel blockers than β-blockers or ACE inhibitors. It has been suggested that hypertension in African-Americans is not as angiotensin II-dependent as it appears to be in Caucasians.
An asymptomatic 3-year-old male presents for a routine check-up. On examination you notice a systolic heart murmur. It is heard best in the lower precordium and has a low, short tone similar to a plucked string or kazoo. It does not radiate to the axillae or the back and seems to decrease with inspiration. The remainder of the examination is normal. What is the most likely diagnosis?
Still’s murmur

There are several benign murmurs of childhood that have no association with physiologic or anatomic abnormalities. Of these, Still’s murmur best fits the murmur described. The cause of Still’s murmur is unknown, but it may be due to vibrations in the chordae tendinae, semilunar valves, or ventricular wall.